Logic-Compactness in complete lattice - MathOverflow most recent 30 from mathoverflow.net 2025-12-19T04:18:43Z https://mathoverflow.net/feeds/question/506003 https://creativecommons.org/licenses/by-sa/4.0/rdf https://mathoverflow.net/q/506003 3 Logic-Compactness in complete lattice Florian https://mathoverflow.net/users/581879 2025-12-17T03:13:16Z 2025-12-19T01:13:29Z <p>Assume <span class="math-container">$\Gamma$</span> is an infinite set composed of formulas (of finite length), and <span class="math-container">$A$</span> is a formula (of finite length). For example, <span class="math-container">$\Gamma=\{x,y\sqcup z,x_1,x_2,x_3,\cdots\}$</span>, <span class="math-container">$A=(x\sqcap y)\sqcup(x\sqcap z)$</span>. Obviously, <span class="math-container">$\mathop⨅\Gamma\le A$</span> holds in any of the complete distributive lattices <span class="math-container">$\langle L,\le\rangle$</span> (i.e. <span class="math-container">$\mathop⨅\Gamma\le A$</span> holds for any <span class="math-container">$x,y,z,x_1,x_2,x_3,\cdots\in L$</span>). Meanwhile, we also can find a finite subset <span class="math-container">$\{x,y\sqcup z\}\subseteq\Gamma$</span> such that <span class="math-container">$\mathop⨅\{x,y\sqcup z\}\le A$</span> holds in any of the complete distributive lattices <span class="math-container">$\langle L,\le\rangle$</span> (i.e.<span class="math-container">$\mathop⨅\{x,y\sqcup z\}\le A$</span> holds for any <span class="math-container">$x,y,z\in L$</span>).</p> <p>Let <span class="math-container">$\mathcal{C}$</span> be a kind of complete lattice (with extra operations). <span class="math-container">$\mathcal{C}$</span> is <em><strong>logic-compact</strong></em> iff <span class="math-container">$\mathcal{C}$</span> satisfies the following property:</p> <blockquote> <p>For any infinite set <span class="math-container">$\Gamma$</span> composed of formulas (of finite length), for any formula <span class="math-container">$A$</span> (of finite length), if <span class="math-container">$\mathop⨅\Gamma\le A$</span> holds in any instance <span class="math-container">$\mathcal{L}$</span> of <span class="math-container">$\mathcal{C}$</span>, then we can always find a finite subset <span class="math-container">$\Gamma_0\subseteq\Gamma$</span> such that <span class="math-container">$\mathop⨅\Gamma_0\le A$</span> holds in any instance <span class="math-container">$\mathcal{L}$</span> of <span class="math-container">$\mathcal{C}$</span>. (The operators in the formulas mentioned above contain both those in the lattice and extra equipped.)</p> </blockquote> <p>Another example: the infinite inequality <span class="math-container">$x\sqcap(x\to y)\sqcap x_1\sqcap x_2\sqcap x_3\cdots\le y$</span> holds in any of the Heyting algebras, which can be attributed to the finite inequality <span class="math-container">$x\sqcap(x\to y)\le y$</span> holding in any of the Heyting algebras. I'm wondering whether any infinite inequality holding in any of the algebra can be attributed to a finite inequality holding in any of the algebra.</p> <p>So, my question is: Does any kind of complete lattice with any operation have <em><strong>logic-compactness</strong></em>? If not, what conditions do the complete lattice and the operations need to satisfy for having the property? (e.g., bounded, distributive, etc.)</p> https://mathoverflow.net/questions/506003/-/506012#506012 5 Answer by Emil Jeřábek for Logic-Compactness in complete lattice Emil Jeřábek https://mathoverflow.net/users/12705 2025-12-17T09:34:55Z 2025-12-17T19:54:41Z <p>While the question is still unclear, it seems now that it is very different from the question answered below, which was based on my best guess from reading the first revision.</p> <hr /> <p>Since MathJax does not seem to support <code>\bigsqcap</code> anyway, I’ll write meets using the standard <span class="math-container">$\bigwedge$</span> notation.</p> <p>Logic compactness is essentially equivalent to well foundedness:</p> <blockquote> <p><strong>Proposition.</strong> Let <span class="math-container">$L$</span> be a lattice.</p> <ol> <li>If <span class="math-container">$L$</span> is well founded, then for every nonempty <span class="math-container">$\Gamma\subseteq L$</span>, <span class="math-container">$\bigwedge\Gamma$</span> exists, and equals <span class="math-container">$\bigwedge\Gamma_0$</span> for some finite <span class="math-container">$\Gamma_0\subseteq\Gamma$</span>. Thus, <span class="math-container">$L$</span> is logic-compact.</li> <li>If <span class="math-container">$\bigwedge\Gamma$</span> exists for every nonempty <span class="math-container">$\Gamma\subseteq L$</span>, and <span class="math-container">$L$</span> is logic-compact, then <span class="math-container">$L$</span> is well founded.</li> <li>If <span class="math-container">$\bigwedge\Gamma$</span> does not necessarily exist, then I do not know how to interpret the definition of logic compactness in the first place.</li> </ol> </blockquote> <p><strong>Proof:</strong></p> <ol> <li><p><span class="math-container">$\{\bigwedge\Gamma_0:\varnothing\ne\Gamma_0\subseteq\Gamma\text{ finite}\}\subseteq L$</span> is nonempty, hence it has a minimal element <span class="math-container">$\bigwedge\Gamma_0$</span> by well foundedness. Then <span class="math-container">$\bigwedge\Gamma_0$</span> is <span class="math-container">$\bigwedge\Gamma$</span>, i.e., <span class="math-container">$\bigwedge\Gamma_0\le x$</span> for each <span class="math-container">$x\in\Gamma$</span>, lest <span class="math-container">$\bigwedge(\Gamma_0\cup\{x\})&lt;\bigwedge\Gamma_0$</span>, contradicting minimality.</p> </li> <li><p>If <span class="math-container">$L$</span> is not well founded, there is a strictly decreasing chain <span class="math-container">$x_0&gt;x_1&gt;x_2&gt;\cdots$</span>. Put <span class="math-container">$\Gamma=\{x_n:n\in\omega\}$</span> and <span class="math-container">$y=\bigwedge\Gamma$</span>. Then <span class="math-container">$y&lt;\bigwedge\Gamma_0=\min\Gamma_0\in\Gamma$</span> for any nonempty finite <span class="math-container">$\Gamma_0\subseteq\Gamma$</span>, witnessing that <span class="math-container">$L$</span> is not logic-compact.</p> </li> <li><p>By fiat.</p> </li> </ol>